Alissa: If, as the mayor says, the city can no longer continue to fund both the children's museum and local children'...

RC on February 9 at 02:29AM

Please explain

I would appreciate a break down of how to reach the correct answer and why the other answers are incorrect. Thank you!

Replies
Create a free account to read and take part in forum discussions.

Already have an account? log in

Emil-Kunkin on February 10 at 04:09AM

Both parties are discussing what should be done if the city can only afford to keep one of the options. A thinks the city should keep the museum, since it is interactive. G thinks they should keep the TV program since it has a wider reach. Our job is to find something they must disagree about.

We don't know either of their positions on A. They both are arguing what would happen in the hypothetical that the city must choose between the two.

Similarly, we don't know either of their positions on the truthfulness of the mayors statements.

We know both of their positions on C. A Supports killing the TV program if it means saving the museum, and G thinks the opposite. This is the correct answer.

D is incorrect because we don't know As position on the matter. While g would agree, a could also agree and simply think that interactivity is more important than reach.

Similarly E is incorrect because we don't know gs position on it. G could simply value wide reach more.

RC on February 12 at 07:22PM

Thank you for taking the time to explain that, Emil!